The student could take which one of the following groups of courses during the summer school session?

brian.eugene.smith@gmail.com on December 28, 2022

Setup

Hello, For the setup, I have a few questions 1) I don't understand why she chooses to look what happens if H, M, & W aren't selected. Put another way, how do we know to focus on P, S, & T? 2) I don't understand how she is able to deduct that we have to have at least 2 blanks in the out group. By the rules I show we should have at least 3 S&M and/or H, P&T and/or M, and P&S and/or W. Will you please explain this more?

Reply
Create a free account to read and take part in forum discussions.

Already have an account? log in

Emil-Kunkin on January 24, 2023

Hi, while I don't have a great answer to your first question (I can't seem to access the video, but I suspect that would be because she was looking at the impact of what would happen if our conditional rules were not in force in addition to what would happen if they were in force), I think that for the second question that was an intermediate deduction on the way to showing there have to be three out.